Range of a Matrix Transformation linear algebra

Click For Summary
The discussion centers on determining the conditions under which the matrix equation Ax = b has solutions for the given matrix A and vector b. Initially, there was confusion regarding the row reduction process, but it was clarified that the correct form of the augmented matrix reveals that not all b in R3 yield a solution. Specifically, a solution exists if the equation b_3 - 5b_1 - 14b_2 = 0 holds, indicating a dependency among the components of b. The geometric interpretation suggests that the solutions form a plane in R3, parameterized by b_1 and b_2. Thus, the set of b for which Ax = b has a solution is characterized by this linear relationship among its components.
SpiffyEh
Messages
191
Reaction score
0

Homework Statement


Given,
A =
[ 1 -3 4;
-3 2 6;
5 -1 -8]

b =
[b_1;
b_2;
b_3]
Show that there does not exist a solution to Ax = b for every b in R3 and describe the set of all {b1,b2,b3} for which Ax = b does have a solution.


Homework Equations


row reduction


The Attempt at a Solution


I row reduced until I got the following augmented matrix:
[ 1 -3 4 | b_1;
0 1 (-18/7)| (b_2 + 3*b_1)/7;
0 0 8 | b_3-5*b_1 - 14*b_2]
I'm confused about this because I was lead to believe that since there is a pivot for every value then there should exist a solution for every b in R3. And There is in fact a pivot in every row. Can someone explain to me if there is a solution or not and why please. I'm just not seeing why there wouldn't be one. Thank you
 
Physics news on Phys.org
Are you sure you wrote down the matrix correctly? In particular, is the 4 in the first row supposed to be a -4?
 
vela said:
Are you sure you wrote down the matrix correctly? In particular, is the 4 in the first row supposed to be a -4?

oops, wow I feel dumb. I even double checked it. So, I had the right idea at least right?
 
ok so with that -4 in place I got the matrix to be:

[ 1 -3 -4 | b_1;
0 1 (6/7)| (b_2 + 3*b_1)/7;
0 0 0 | b_3-5*b_1 - 14*b_2]

from this i can see that there isn't an answer for all b in R3. The 2nd part of the question asks "describe the set of all {b1,b2,b3} for which Ax = b does have a solution"

this would be where b_3-5*b_1 - 14*b_2 is equal to zero. Otherwise there would be no solution. Would I just write this or do i need to say something about b1 and b2? Would i need to say that b1 and b2 could be anything? Or how would I go about saying that?
 
It depends on what "describe" means here. You might be expected to give a geometric interpretation of the solution set.
 
SpiffyEh said:
ok so with that -4 in place I got the matrix to be:

[ 1 -3 -4 | b_1;
0 1 (6/7)| (b_2 + 3*b_1)/7;
0 0 0 | b_3-5*b_1 - 14*b_2]

from this i can see that there isn't an answer for all b in R3. The 2nd part of the question asks "describe the set of all {b1,b2,b3} for which Ax = b does have a solution"

this would be where b_3-5*b_1 - 14*b_2 is equal to zero. Otherwise there would be no solution. Would I just write this or do i need to say something about b1 and b2? Would i need to say that b1 and b2 could be anything? Or how would I go about saying that?
Good: b_3- 5b_1- 14b_2= 0 and you can write that as b_3= 5b_1+ 14b_2 so that
\begin{pmatrix}b_1 \\ b_2 \\ b_3\end{pmatrix}= \begin{pmatrix} b_1 \\ b_2 \\ 5b_1+ 14b_2 \end{pmatrix}= b_1\begin{pmatrix}1 \\ 0 \\ 5\end{pmatrix}+ b_2\begin{pmatrix}0 \\ 1 \\ 14\end{pmatrix}.

Geometrically, what is that?
 
Question: A clock's minute hand has length 4 and its hour hand has length 3. What is the distance between the tips at the moment when it is increasing most rapidly?(Putnam Exam Question) Answer: Making assumption that both the hands moves at constant angular velocities, the answer is ## \sqrt{7} .## But don't you think this assumption is somewhat doubtful and wrong?

Similar threads

  • · Replies 5 ·
Replies
5
Views
1K
  • · Replies 10 ·
Replies
10
Views
3K
Replies
8
Views
2K
  • · Replies 4 ·
Replies
4
Views
3K
  • · Replies 4 ·
Replies
4
Views
2K
  • · Replies 3 ·
Replies
3
Views
1K
  • · Replies 7 ·
Replies
7
Views
2K
  • · Replies 2 ·
Replies
2
Views
2K
Replies
12
Views
2K
  • · Replies 24 ·
Replies
24
Views
4K